Proving by Contrapositive (specific question within)











up vote
0
down vote

favorite
2












I'm having issues coming up with a contrapositive proof for the following question.



As far as I know, a proof by contraposition is based on the following :




$overline Q to overline P equiv P to Q$




or that's where I'm mistaken?



The question is:




X,Y,Z are natural numbers.
if $X^3+Y^3=Z^3$ , then at least one of them is divisible by 3.
Provide a proof by contrapositive.




I've tried to substitude X,Y,Z with (3A-1),(3B-1),(3C-1) , essentially trying to get to a point where:



$overline Q$ : none is divisble by 3 (3N-1).



$overline P$ : $X^3+Y^3neq Z^3$



$overline Q$ -> $overline P$ $equiv$ P -> Q



I tried to simplify the equation but it doesn't look like I got anywhere, where am I being wrong?



Thanks for helping!










share|cite|improve this question




















  • 1




    You assumed that a number not divisible by $3$ must be of the form $3N-1$. What about $4$ ?
    – Mauro ALLEGRANZA
    Nov 15 at 14:07










  • @MauroALLEGRANZA, I should have written it specifically, my professor gave me that idea, what other route can I go to solve this? Thanks for the quick reply.
    – Idan Botbol
    Nov 15 at 14:10












  • I think if you consider x,y,z individually in form that is not divisible by 3 i.e., 3n-1 and 3n-2, you would have total 27 permutations of these numbers and you would have 27 cases to prove. So I think you should think of a different approach.
    – Jimmy
    Nov 15 at 14:16










  • Have you learned about the modulo notation in your course?
    – F.Carette
    Nov 15 at 14:18






  • 1




    Maybe this will help. You can write $X^3=Xmod(3)$, whenever $X$ is not divisible by 3.
    – Jimmy
    Nov 15 at 15:13















up vote
0
down vote

favorite
2












I'm having issues coming up with a contrapositive proof for the following question.



As far as I know, a proof by contraposition is based on the following :




$overline Q to overline P equiv P to Q$




or that's where I'm mistaken?



The question is:




X,Y,Z are natural numbers.
if $X^3+Y^3=Z^3$ , then at least one of them is divisible by 3.
Provide a proof by contrapositive.




I've tried to substitude X,Y,Z with (3A-1),(3B-1),(3C-1) , essentially trying to get to a point where:



$overline Q$ : none is divisble by 3 (3N-1).



$overline P$ : $X^3+Y^3neq Z^3$



$overline Q$ -> $overline P$ $equiv$ P -> Q



I tried to simplify the equation but it doesn't look like I got anywhere, where am I being wrong?



Thanks for helping!










share|cite|improve this question




















  • 1




    You assumed that a number not divisible by $3$ must be of the form $3N-1$. What about $4$ ?
    – Mauro ALLEGRANZA
    Nov 15 at 14:07










  • @MauroALLEGRANZA, I should have written it specifically, my professor gave me that idea, what other route can I go to solve this? Thanks for the quick reply.
    – Idan Botbol
    Nov 15 at 14:10












  • I think if you consider x,y,z individually in form that is not divisible by 3 i.e., 3n-1 and 3n-2, you would have total 27 permutations of these numbers and you would have 27 cases to prove. So I think you should think of a different approach.
    – Jimmy
    Nov 15 at 14:16










  • Have you learned about the modulo notation in your course?
    – F.Carette
    Nov 15 at 14:18






  • 1




    Maybe this will help. You can write $X^3=Xmod(3)$, whenever $X$ is not divisible by 3.
    – Jimmy
    Nov 15 at 15:13













up vote
0
down vote

favorite
2









up vote
0
down vote

favorite
2






2





I'm having issues coming up with a contrapositive proof for the following question.



As far as I know, a proof by contraposition is based on the following :




$overline Q to overline P equiv P to Q$




or that's where I'm mistaken?



The question is:




X,Y,Z are natural numbers.
if $X^3+Y^3=Z^3$ , then at least one of them is divisible by 3.
Provide a proof by contrapositive.




I've tried to substitude X,Y,Z with (3A-1),(3B-1),(3C-1) , essentially trying to get to a point where:



$overline Q$ : none is divisble by 3 (3N-1).



$overline P$ : $X^3+Y^3neq Z^3$



$overline Q$ -> $overline P$ $equiv$ P -> Q



I tried to simplify the equation but it doesn't look like I got anywhere, where am I being wrong?



Thanks for helping!










share|cite|improve this question















I'm having issues coming up with a contrapositive proof for the following question.



As far as I know, a proof by contraposition is based on the following :




$overline Q to overline P equiv P to Q$




or that's where I'm mistaken?



The question is:




X,Y,Z are natural numbers.
if $X^3+Y^3=Z^3$ , then at least one of them is divisible by 3.
Provide a proof by contrapositive.




I've tried to substitude X,Y,Z with (3A-1),(3B-1),(3C-1) , essentially trying to get to a point where:



$overline Q$ : none is divisble by 3 (3N-1).



$overline P$ : $X^3+Y^3neq Z^3$



$overline Q$ -> $overline P$ $equiv$ P -> Q



I tried to simplify the equation but it doesn't look like I got anywhere, where am I being wrong?



Thanks for helping!







discrete-mathematics logic proof-explanation






share|cite|improve this question















share|cite|improve this question













share|cite|improve this question




share|cite|improve this question








edited Nov 15 at 14:04









Mauro ALLEGRANZA

63.9k448110




63.9k448110










asked Nov 15 at 14:00









Idan Botbol

6




6








  • 1




    You assumed that a number not divisible by $3$ must be of the form $3N-1$. What about $4$ ?
    – Mauro ALLEGRANZA
    Nov 15 at 14:07










  • @MauroALLEGRANZA, I should have written it specifically, my professor gave me that idea, what other route can I go to solve this? Thanks for the quick reply.
    – Idan Botbol
    Nov 15 at 14:10












  • I think if you consider x,y,z individually in form that is not divisible by 3 i.e., 3n-1 and 3n-2, you would have total 27 permutations of these numbers and you would have 27 cases to prove. So I think you should think of a different approach.
    – Jimmy
    Nov 15 at 14:16










  • Have you learned about the modulo notation in your course?
    – F.Carette
    Nov 15 at 14:18






  • 1




    Maybe this will help. You can write $X^3=Xmod(3)$, whenever $X$ is not divisible by 3.
    – Jimmy
    Nov 15 at 15:13














  • 1




    You assumed that a number not divisible by $3$ must be of the form $3N-1$. What about $4$ ?
    – Mauro ALLEGRANZA
    Nov 15 at 14:07










  • @MauroALLEGRANZA, I should have written it specifically, my professor gave me that idea, what other route can I go to solve this? Thanks for the quick reply.
    – Idan Botbol
    Nov 15 at 14:10












  • I think if you consider x,y,z individually in form that is not divisible by 3 i.e., 3n-1 and 3n-2, you would have total 27 permutations of these numbers and you would have 27 cases to prove. So I think you should think of a different approach.
    – Jimmy
    Nov 15 at 14:16










  • Have you learned about the modulo notation in your course?
    – F.Carette
    Nov 15 at 14:18






  • 1




    Maybe this will help. You can write $X^3=Xmod(3)$, whenever $X$ is not divisible by 3.
    – Jimmy
    Nov 15 at 15:13








1




1




You assumed that a number not divisible by $3$ must be of the form $3N-1$. What about $4$ ?
– Mauro ALLEGRANZA
Nov 15 at 14:07




You assumed that a number not divisible by $3$ must be of the form $3N-1$. What about $4$ ?
– Mauro ALLEGRANZA
Nov 15 at 14:07












@MauroALLEGRANZA, I should have written it specifically, my professor gave me that idea, what other route can I go to solve this? Thanks for the quick reply.
– Idan Botbol
Nov 15 at 14:10






@MauroALLEGRANZA, I should have written it specifically, my professor gave me that idea, what other route can I go to solve this? Thanks for the quick reply.
– Idan Botbol
Nov 15 at 14:10














I think if you consider x,y,z individually in form that is not divisible by 3 i.e., 3n-1 and 3n-2, you would have total 27 permutations of these numbers and you would have 27 cases to prove. So I think you should think of a different approach.
– Jimmy
Nov 15 at 14:16




I think if you consider x,y,z individually in form that is not divisible by 3 i.e., 3n-1 and 3n-2, you would have total 27 permutations of these numbers and you would have 27 cases to prove. So I think you should think of a different approach.
– Jimmy
Nov 15 at 14:16












Have you learned about the modulo notation in your course?
– F.Carette
Nov 15 at 14:18




Have you learned about the modulo notation in your course?
– F.Carette
Nov 15 at 14:18




1




1




Maybe this will help. You can write $X^3=Xmod(3)$, whenever $X$ is not divisible by 3.
– Jimmy
Nov 15 at 15:13




Maybe this will help. You can write $X^3=Xmod(3)$, whenever $X$ is not divisible by 3.
– Jimmy
Nov 15 at 15:13










1 Answer
1






active

oldest

votes

















up vote
0
down vote













Following the comments, if we have $X, Y$ and $Z$ none of which are divisible by 3 and such that $X^3 + Y^3 = Z^3$, we may assume that $X,Y equiv 1$ (mod 3) and $Z equiv 2$ (mod 3). So, we may assume that there are $a$ $b$ and $c$ such that $X = 3a+1$, $Y = 3b+1$ and $Z = 3c+2$. By applying the binomial expansion to $X^3$ and $Y^3$, it turns out that $X^3 equiv 1$ (mod 9) and $Y^3 equiv 1$ (mod 9), so that $X^3 + Y^3 equiv 2$ (mod 9). But if you apply the binomial expansion to $Z^3 = (3c+2)^3$, it turns out that $Z^3 equiv 8$ (mod 9). Thus, the equality $X^3 + Y^3 = Z^3$ cannot hold.






share|cite|improve this answer





















    Your Answer





    StackExchange.ifUsing("editor", function () {
    return StackExchange.using("mathjaxEditing", function () {
    StackExchange.MarkdownEditor.creationCallbacks.add(function (editor, postfix) {
    StackExchange.mathjaxEditing.prepareWmdForMathJax(editor, postfix, [["$", "$"], ["\\(","\\)"]]);
    });
    });
    }, "mathjax-editing");

    StackExchange.ready(function() {
    var channelOptions = {
    tags: "".split(" "),
    id: "69"
    };
    initTagRenderer("".split(" "), "".split(" "), channelOptions);

    StackExchange.using("externalEditor", function() {
    // Have to fire editor after snippets, if snippets enabled
    if (StackExchange.settings.snippets.snippetsEnabled) {
    StackExchange.using("snippets", function() {
    createEditor();
    });
    }
    else {
    createEditor();
    }
    });

    function createEditor() {
    StackExchange.prepareEditor({
    heartbeatType: 'answer',
    convertImagesToLinks: true,
    noModals: true,
    showLowRepImageUploadWarning: true,
    reputationToPostImages: 10,
    bindNavPrevention: true,
    postfix: "",
    imageUploader: {
    brandingHtml: "Powered by u003ca class="icon-imgur-white" href="https://imgur.com/"u003eu003c/au003e",
    contentPolicyHtml: "User contributions licensed under u003ca href="https://creativecommons.org/licenses/by-sa/3.0/"u003ecc by-sa 3.0 with attribution requiredu003c/au003e u003ca href="https://stackoverflow.com/legal/content-policy"u003e(content policy)u003c/au003e",
    allowUrls: true
    },
    noCode: true, onDemand: true,
    discardSelector: ".discard-answer"
    ,immediatelyShowMarkdownHelp:true
    });


    }
    });














    draft saved

    draft discarded


















    StackExchange.ready(
    function () {
    StackExchange.openid.initPostLogin('.new-post-login', 'https%3a%2f%2fmath.stackexchange.com%2fquestions%2f2999739%2fproving-by-contrapositive-specific-question-within%23new-answer', 'question_page');
    }
    );

    Post as a guest















    Required, but never shown

























    1 Answer
    1






    active

    oldest

    votes








    1 Answer
    1






    active

    oldest

    votes









    active

    oldest

    votes






    active

    oldest

    votes








    up vote
    0
    down vote













    Following the comments, if we have $X, Y$ and $Z$ none of which are divisible by 3 and such that $X^3 + Y^3 = Z^3$, we may assume that $X,Y equiv 1$ (mod 3) and $Z equiv 2$ (mod 3). So, we may assume that there are $a$ $b$ and $c$ such that $X = 3a+1$, $Y = 3b+1$ and $Z = 3c+2$. By applying the binomial expansion to $X^3$ and $Y^3$, it turns out that $X^3 equiv 1$ (mod 9) and $Y^3 equiv 1$ (mod 9), so that $X^3 + Y^3 equiv 2$ (mod 9). But if you apply the binomial expansion to $Z^3 = (3c+2)^3$, it turns out that $Z^3 equiv 8$ (mod 9). Thus, the equality $X^3 + Y^3 = Z^3$ cannot hold.






    share|cite|improve this answer

























      up vote
      0
      down vote













      Following the comments, if we have $X, Y$ and $Z$ none of which are divisible by 3 and such that $X^3 + Y^3 = Z^3$, we may assume that $X,Y equiv 1$ (mod 3) and $Z equiv 2$ (mod 3). So, we may assume that there are $a$ $b$ and $c$ such that $X = 3a+1$, $Y = 3b+1$ and $Z = 3c+2$. By applying the binomial expansion to $X^3$ and $Y^3$, it turns out that $X^3 equiv 1$ (mod 9) and $Y^3 equiv 1$ (mod 9), so that $X^3 + Y^3 equiv 2$ (mod 9). But if you apply the binomial expansion to $Z^3 = (3c+2)^3$, it turns out that $Z^3 equiv 8$ (mod 9). Thus, the equality $X^3 + Y^3 = Z^3$ cannot hold.






      share|cite|improve this answer























        up vote
        0
        down vote










        up vote
        0
        down vote









        Following the comments, if we have $X, Y$ and $Z$ none of which are divisible by 3 and such that $X^3 + Y^3 = Z^3$, we may assume that $X,Y equiv 1$ (mod 3) and $Z equiv 2$ (mod 3). So, we may assume that there are $a$ $b$ and $c$ such that $X = 3a+1$, $Y = 3b+1$ and $Z = 3c+2$. By applying the binomial expansion to $X^3$ and $Y^3$, it turns out that $X^3 equiv 1$ (mod 9) and $Y^3 equiv 1$ (mod 9), so that $X^3 + Y^3 equiv 2$ (mod 9). But if you apply the binomial expansion to $Z^3 = (3c+2)^3$, it turns out that $Z^3 equiv 8$ (mod 9). Thus, the equality $X^3 + Y^3 = Z^3$ cannot hold.






        share|cite|improve this answer












        Following the comments, if we have $X, Y$ and $Z$ none of which are divisible by 3 and such that $X^3 + Y^3 = Z^3$, we may assume that $X,Y equiv 1$ (mod 3) and $Z equiv 2$ (mod 3). So, we may assume that there are $a$ $b$ and $c$ such that $X = 3a+1$, $Y = 3b+1$ and $Z = 3c+2$. By applying the binomial expansion to $X^3$ and $Y^3$, it turns out that $X^3 equiv 1$ (mod 9) and $Y^3 equiv 1$ (mod 9), so that $X^3 + Y^3 equiv 2$ (mod 9). But if you apply the binomial expansion to $Z^3 = (3c+2)^3$, it turns out that $Z^3 equiv 8$ (mod 9). Thus, the equality $X^3 + Y^3 = Z^3$ cannot hold.







        share|cite|improve this answer












        share|cite|improve this answer



        share|cite|improve this answer










        answered Nov 23 at 17:51









        Tony Dean

        512




        512






























            draft saved

            draft discarded




















































            Thanks for contributing an answer to Mathematics Stack Exchange!


            • Please be sure to answer the question. Provide details and share your research!

            But avoid



            • Asking for help, clarification, or responding to other answers.

            • Making statements based on opinion; back them up with references or personal experience.


            Use MathJax to format equations. MathJax reference.


            To learn more, see our tips on writing great answers.





            Some of your past answers have not been well-received, and you're in danger of being blocked from answering.


            Please pay close attention to the following guidance:


            • Please be sure to answer the question. Provide details and share your research!

            But avoid



            • Asking for help, clarification, or responding to other answers.

            • Making statements based on opinion; back them up with references or personal experience.


            To learn more, see our tips on writing great answers.




            draft saved


            draft discarded














            StackExchange.ready(
            function () {
            StackExchange.openid.initPostLogin('.new-post-login', 'https%3a%2f%2fmath.stackexchange.com%2fquestions%2f2999739%2fproving-by-contrapositive-specific-question-within%23new-answer', 'question_page');
            }
            );

            Post as a guest















            Required, but never shown





















































            Required, but never shown














            Required, but never shown












            Required, but never shown







            Required, but never shown

































            Required, but never shown














            Required, but never shown












            Required, but never shown







            Required, but never shown







            Popular posts from this blog

            Le Mesnil-Réaume

            Ida-Boy-Ed-Garten

            web3.py web3.isConnected() returns false always